1
$\begingroup$

Currently, I am reading a paper about the Gaussian Process in Neural Network [1]. In the solution of the main result in this paper, the author applied Stein's lemma and claimed an equation about the partial derivative of expectation as below ([1], page. 68),

Let $\Psi(w,\tau) = \mathbb{E}_{z \sim \mathcal{N}(0,1)}\psi(g_1,\dots,g_{m-1},w+\tau z)$. By Stein's Lemma, $\Psi(w,\tau)$ is differentiable in $w$, and $$\partial_{w}\Psi(w,\tau) = \tau^{-1}\mathbb{E}_{z \sim \mathcal{N}(0,1)}[z\psi(g_1,\dots,g_{m-1},w+\tau z)],$$

Stein's Lemma: For jointly Gaussian random variable $Z_1$ and $Z_2$ with zero mean, and any function $\phi:\mathcal{R} \rightarrow \mathcal{R}$, where $\mathbb{E}[\phi'(Z_1)]$ and $\mathbb{E}[Z_1\phi(Z_2)]$ exists, we have \begin{equation} \mathbb{E}[Z_1 \phi(Z_2)] = Cov(Z_1,Z_2) \mathbb{E}[\phi'(Z_2)], \end{equation} where $Cov(Z_1,Z_2)$ is the covariance of $Z_1$ and $Z_2$.

As I understand, the author applied Stein's lemma for $z$ and $\phi(g_1,\dots,g_{m-1},w+\tau z)$ to obtain $$\mathbb{E}[z\phi(g_1,\dots,g_{m-1},w+\tau z)] = \tau \mathbb{E}[\phi'(g_1,\dots,g_{m-1},w+\tau z)].$$

However, I couldn't figure out why the author can obtain the equation of the partial derivative in terms of $w$ as above. I am very appreciative if someone explains it to me.

[1]. https://arxiv.org/abs/1910.12478

$\endgroup$

1 Answer 1

2
$\begingroup$

Stein's lemma (with $Z_1=Z_2$) gives $$\mathbb{E}[z\psi(g_1,\dots,g_{m-1},w+\tau z)] =\text{cov}\,(z,z)\mathbb{E}[\frac{d}{dz}\psi(g_1,\dots,g_{m-1},w+\tau z)]$$ $$\qquad=\tau\frac{d}{dw}\mathbb{E}[\psi(g_1,\dots,g_{m-1},w+\tau z)]\equiv \tau\partial_w\Psi(w,\tau),$$ since $\text{cov}\,(z,z)=1$ and $df(w+\tau z)/dz=\tau df(w+\tau z)/dw$.

$\endgroup$
1
  • 1
    $\begingroup$ @Quick Exactly as Carlo Beenakker says. You turned the 2 steps around. You do not first get $\partial_w$ derivative and then use that ``Stein's lemma.'' Rather you use the lemma to get the $\partial_w$ derivative formula you first wrote, exactly as Carlo Beenakker has showed. (I just wanted to add to their answer that you switched the 2 steps.) $\endgroup$ Jan 6, 2022 at 18:53

Your Answer

By clicking “Post Your Answer”, you agree to our terms of service and acknowledge you have read our privacy policy.

Not the answer you're looking for? Browse other questions tagged or ask your own question.